Free AANP Practice Questions

Take our free AANP practice test, Qbank, Test 6. These exam questions include answers and detailed explanations.

AANP Exam Questions

AANP Question 1: A 45-year-old female patient presents with symptoms of excessive thirst, frequent urination, and unexplained weight loss. She has a family history of diabetes mellitus. On examination, her blood pressure is normal, but her fasting blood glucose level is elevated. Which of the following endocrine disorders is most likely responsible for her symptoms?
A) Hyperthyroidism
B) Hypoparathyroidism
C) Cushing’s syndrome
D) Diabetes mellitus type 2
E) Addison’s disease

Rationale: This question assesses the nurse practitioner’s ability to recognize common symptoms of diabetes mellitus and differentiate it from other endocrine disorders. The patient’s age, family history, elevated fasting blood glucose level, and symptoms of polydipsia, polyuria, and weight loss are indicative of diabetes mellitus type 2.

200 Quizzes for Family Nurse Practitioners qbankproacademy.com

AANP Practice Questions QBank, Test 2
AANP Practice Questions QBank, Test 3
AANP Practice Questions QBank, Test 4
AANP Practice Questions QBank, Test 5

 

@qbankproacademy

4000+ Free NCLEX QUESTIONS. Go to QBankProAcademy.com FREE Qbank questions for NCLEX RN, PN, HESI Exit, Med Surg, AANP, ANCC, and HesiA2. Practice questions, quizzes, and listen to the Free Podcast. In this video, we review a question on Aging in Nursing Care. Free nursing NCLEX 60-DAY CHALLENGE https://qbankpro.thinkific.com/courses/qbankpro-academy At QbankproAcademy.com our mission is to provide free QBanks, videos, and the most up to date test prep information for nurses. If you find our website helpful, please tell other aspiring nurses, nursing students, and professors. Please link to our site from your blogs, videos, and college websites, or share us on your favorite social media sites. Thank you for your support! nclex Nurses Nursing aanp qbank ancc hesi medsurg qbank

♬ original sound – QBank Pro Academy – QBank Pro Academy

 

AANP Question 2: A 35-year-old male patient presents with a chief complaint of fatigue, muscle weakness, and frequent muscle cramps. On physical examination, the nurse practitioner notes decreased muscle tone and decreased deep tendon reflexes. Laboratory results show low serum calcium levels and an elevated parathyroid hormone (PTH) level. What endocrine disorder is most likely causing these findings?
A) Hyperthyroidism
B) Hypoparathyroidism
C) Acromegaly
D) Cushing’s syndrome
E) Pheochromocytoma

 

Rationale: This question tests the nurse practitioner’s knowledge of the clinical presentation and diagnostic findings associated with hypoparathyroidism. The symptoms of muscle weakness, muscle cramps, decreased muscle tone, and low serum calcium levels are indicative of hypoparathyroidism, which is characterized by insufficient secretion of parathyroid hormone.

AANP, ANCC, nclex, questions and answers, hesi exit
Pancreatitis, anatomy of the pancreas

AANP Question 3: A 28-year-old female patient presents with irregular menstrual periods, hirsutism (excessive facial and body hair), and acne. She reports a history of obesity and insulin resistance. On physical examination, there is evidence of acanthosis nigricans (dark, thickened skin folds). Which of the following endocrine disorders is most likely contributing to her symptoms?
A) Hyperthyroidism
B) Polycystic ovary syndrome (PCOS)
C) Cushing’s syndrome
D) Hypothyroidism
E) Hyperparathyroidism

Your Answer:
QBankProAcademy.com

Rationale: This question evaluates the nurse practitioner’s ability to recognize the clinical manifestations of PCOS, a common endocrine disorder in women of reproductive age. The patient’s symptoms of irregular menstrual periods, hirsutism, acne, obesity, insulin resistance, and acanthosis nigricans are characteristic of PCOS, which is associated with hormonal imbalances, particularly elevated androgens.

AANP Question 4: A 55-year-old male patient presents with complaints of excessive thirst, frequent urination, and fatigue. His fasting blood glucose level is 180 mg/dL. He has no family history of diabetes. Which of the following endocrine disorders is most likely responsible for his symptoms?
A) Hyperthyroidism
B) Hypoparathyroidism
C) Diabetes mellitus type 1
D) Addison’s disease
E) Hypothyroidism

Your Answer:
QBankProAcademy.com

Rationale: This question assesses the nurse practitioner’s ability to recognize the symptoms and diagnostic criteria for differentiating between diabetes mellitus type 1 and type 2. The patient’s age, absence of a family history of diabetes, and significantly elevated fasting blood glucose levels are indicative of diabetes mellitus type 1, which results from autoimmune destruction of pancreatic beta cells.

AANP Question 5: A 42-year-old female patient presents with complaints of heat intolerance, weight loss, and palpitations. On examination, you note an enlarged thyroid gland (goiter). Lab results reveal elevated levels of T3 and T4 and low levels of thyroid-stimulating hormone (TSH). What endocrine disorder is most likely responsible for her symptoms?
A) Hypoparathyroidism
B) Graves’ disease
C) Cushing’s syndrome
D) Hypothyroidism
E) Pheochromocytoma

Your Answer:
QBankProAcademy.com

Rationale: This question tests the nurse practitioner’s knowledge of the clinical presentation and diagnostic findings associated with Graves’ disease, an autoimmune thyroid disorder. The symptoms of heat intolerance, weight loss, palpitations, and an enlarged thyroid gland (goiter) are characteristic of Graves’ disease, which is marked by hyperthyroidism and low TSH levels.

AANP Question 6: A 30-year-old female patient presents with amenorrhea (absence of menstruation), galactorrhea (inappropriate lactation), and a headache. Physical examination reveals no evidence of pregnancy, and a neurological examination is unremarkable. Lab results show elevated levels of prolactin. What endocrine disorder is most likely responsible for her symptoms?
A) Hyperthyroidism
B) Polycystic ovary syndrome (PCOS)
C) Cushing’s syndrome
D) Hyperprolactinemia
E) Hypothyroidism

Your Answer:
QBankProAcademy.com

Rationale: This question evaluates the nurse practitioner’s ability to recognize the clinical presentation of hyperprolactinemia, a condition characterized by elevated levels of prolactin. The patient’s symptoms of amenorrhea, galactorrhea, and headache are indicative of hyperprolactinemia, which can result from various causes, including pituitary tumors or medications.

AANP Question 7: A 40-year-old patient with a history of type 2 diabetes presents with persistent high blood glucose levels despite dietary changes and exercise. The healthcare provider decides to initiate a medication to improve glycemic control. Which of the following medications primarily acts by increasing insulin sensitivity and reducing hepatic glucose production?
A) Metformin
B) Glipizide
C) Insulin glargine
D) Exenatide
E) Canagliflozin

Your Answer:
QBankProAcademy.com

Rationale: This question assesses the nurse practitioner’s knowledge of the mechanism of action of common antidiabetic medications. Metformin primarily works by increasing insulin sensitivity in peripheral tissues and reducing hepatic glucose production. It is often the first-line medication for type 2 diabetes management.

AANP, ANCC, NCLEX, questions and answers HESI Exit
Thyroid anatomy

AANP Question 8: A 50-year-old male patient with a history of hypothyroidism has been taking levothyroxine for several years. He presents with complaints of persistent fatigue, weight gain, and constipation. The healthcare provider suspects inadequate thyroid hormone replacement therapy. Which of the following laboratory tests should be used to assess the adequacy of thyroid hormone replacement in this patient?
A) Serum thyroxine (T4) level
B) Serum triiodothyronine (T3) level
C) Thyroid-stimulating hormone (TSH) level
D) Free thyroxine (FT4) level
E) Total triiodothyronine (total T3) level

Your Answer:
QBankProAcademy.com

Rationale: This question evaluates the nurse practitioner’s understanding of thyroid hormone replacement therapy monitoring. TSH levels are the most reliable indicator of thyroid hormone adequacy. Elevated TSH levels suggest inadequate thyroid hormone replacement, as the pituitary is trying to stimulate the thyroid gland to produce more hormone.

AANP Question 9: A 60-year-old postmenopausal female patient is diagnosed with osteoporosis. The healthcare provider recommends an antiresorptive medication to reduce the risk of fractures. Which of the following medications inhibits bone resorption by suppressing osteoclast activity and is commonly used in the treatment of osteoporosis?
A) Alendronate
B) Calcitonin
C) Teriparatide
D) Raloxifene
E) Denosumab

Your Answer:
QBankProAcademy.com

Rationale: This question assesses the nurse practitioner’s knowledge of medications used in the management of osteoporosis. Alendronate is a bisphosphonate medication that inhibits bone resorption by suppressing osteoclast activity, thus reducing the risk of fractures in patients with osteoporosis.

AANP Question 10: A 55-year-old female patient with a known diagnosis of hyperthyroidism presents with persistent symptoms of tachycardia, anxiety, and weight loss despite taking an antithyroid medication. The healthcare provider suspects inadequate response to the current treatment. What is the next appropriate step in managing this patient’s hyperthyroidism?

A) Increase the dosage of the current antithyroid medication. B) Start thyroid hormone replacement therapy. C) Refer the patient for radioactive iodine therapy. D) Perform a thyroidectomy. E) Prescribe a beta-blocker.

Your Answer:
QBankProAcademy.com

Rationale: This question assesses the nurse practitioner’s understanding of managing hyperthyroidism, particularly addressing symptomatic relief. Prescribing a beta-blocker, such as propranolol, is an essential step to alleviate symptoms like tachycardia and anxiety while the underlying thyroid dysfunction is being addressed.

AANP Question 11: A 35-year-old male patient with type 2 diabetes mellitus is experiencing suboptimal glycemic control despite taking metformin and following a diabetic diet. The healthcare provider decides to add a medication that stimulates insulin secretion from pancreatic beta cells. Which of the following medications is most appropriate for this purpose?
A) Sitagliptin
B) Canagliflozin
C) Pioglitazone
D) Acarbose
E) Glucagon-like peptide-1 (GLP-1) agonist

Your Answer:
QBankProAcademy.com

Rationale: This question evaluates the nurse practitioner’s knowledge of antidiabetic medications and their mechanisms of action. Sitagliptin is a dipeptidyl peptidase-4 (DPP-4) inhibitor that stimulates insulin secretion from pancreatic beta cells, helping to improve glycemic control in patients with type 2 diabetes mellitus.

AANP Question 12: A 60-year-old postmenopausal woman with a history of osteoporosis presents for further evaluation and management. She has experienced a vertebral fracture in the past and is at high risk for future fractures. Which medication should be considered for its anabolic effect on bone formation in this patient?
A) Alendronate
B) Calcitonin
C) Teriparatide
D) Raloxifene
E) Denosumab

Your Answer:
QBankProAcademy.com

Rationale: This question assesses the nurse practitioner’s understanding of medications used for osteoporosis management. Teriparatide is a recombinant parathyroid hormone (PTH) analog that has anabolic effects on bone, promoting bone formation. It is indicated for patients at high risk of fractures, such as those with a history of vertebral fractures.

Chest xray, radiography, nclex, nextgen, free questions, AANP, ANCC, NCLEX, questions and answers HESI Exit
Chest X-ray

AANP Question 13: A 28-year-old female patient presents with symptoms of excessive thirst, frequent urination, and unexplained weight loss. The healthcare provider suspects diabetes mellitus and orders a diagnostic test to confirm the diagnosis. Which of the following tests is the most appropriate initial diagnostic test for diabetes mellitus?
A) Thyroid function panel
B) Hemoglobin A1c
C) Serum cortisol level
D) Serum calcium level
E) Serum follicle-stimulating hormone (FSH) level

Your Answer:
QBankProAcademy.com

Rationale: This question evaluates the nurse practitioner’s knowledge of diagnostic tests for diabetes mellitus. Hemoglobin A1c (HbA1c) is a commonly used test to assess long-term glycemic control and diagnose diabetes mellitus. It reflects average blood glucose levels over the past 2-3 months and is a valuable tool for diagnosing and monitoring diabetes.

AANP Question 14: A 50-year-old male patient presents with symptoms of fatigue, muscle weakness, and bone pain. The healthcare provider suspects primary hyperparathyroidism and plans to confirm the diagnosis with laboratory testing. Which of the following laboratory findings is most consistent with primary hyperparathyroidism?
A) Elevated serum thyroxine (T4) level
B) Elevated serum cortisol level
C) Elevated serum testosterone level
D) Elevated serum parathyroid hormone (PTH) level
E) Decreased serum luteinizing hormone (LH) level

Your Answer:
QBankProAcademy.com

Rationale: This question assesses the nurse practitioner’s understanding of the diagnostic criteria for primary hyperparathyroidism. Elevated serum PTH levels in the presence of hypercalcemia are characteristic findings in primary hyperparathyroidism.

AANP Question 15: A 40-year-old female patient presents with amenorrhea (absence of menstruation) and symptoms suggestive of a pituitary disorder, such as visual field disturbances. The healthcare provider suspects a pituitary adenoma and orders diagnostic imaging. What imaging modality is most appropriate for visualizing pituitary adenomas?
A) Chest X-ray
B) Abdominal ultrasound
C) Magnetic resonance imaging (MRI) of the brain
D) Computed tomography (CT) of the chest
E) Bone scan

Your Answer:
QBankProAcademy.com

Rationale: This question tests the nurse practitioner’s knowledge of the preferred imaging modality for visualizing pituitary adenomas. MRI of the brain with contrast is the most appropriate imaging technique for visualizing pituitary adenomas and assessing their size and location within the sella turcica.

AANP Question 16: A 32-year-old male patient presents with fatigue, muscle weakness, and a blood pressure reading of 160/100 mm Hg. The healthcare provider suspects an adrenal disorder and orders diagnostic tests. Which of the following diagnostic tests is the initial screening test for assessing adrenal function?
A) Thyroid function panel
B) Serum cortisol level
C) Serum testosterone level
D) Serum insulin-like growth factor-1 (IGF-1) level
E) Serum parathyroid hormone (PTH) level

Your Answer:
QBankProAcademy.com

AANP Question 17: A 45-year-old female patient presents with symptoms of heat intolerance, weight loss, and palpitations. The healthcare provider suspects hyperthyroidism and plans to order thyroid function tests. Which of the following thyroid function tests provides the best indication?
A) Thyroglobulin
B) Free Iodine
C) TSH
D) Free thyroxine (FT4)
E) Free triiodothyronine (FT3)

Your Answer:
QBankProAcademy.com

Rationale: This question assesses the nurse practitioner’s understanding of thyroid function tests. Thyroid stimulating hormone (TSH) is considered a more accurate reflection of thyroid function than total T4.

AANP Question 18: A 28-year-old female patient presents with excessive thirst, frequent urination, and unexplained weight loss. The healthcare provider suspects diabetes mellitus and plans to order diagnostic tests. Which of the following tests provides information about blood glucose levels over the past 2-3 months and aids in the diagnosis of diabetes?
A) Fasting blood glucose test
B) Oral glucose tolerance test (OGTT)
C) Glycosylated hemoglobin (HbA1c) test
D) Random blood glucose test
E) Serum insulin level

Your Answer:
QBankProAcademy.com

Rationale: This question evaluates the nurse practitioner’s knowledge of diagnostic tests for diabetes mellitus. The HbA1c test provides information about average blood glucose levels over the past 2-3 months and is a valuable tool for diagnosing and monitoring diabetes.

AANP Question 19: A 40-year-old female patient presents with unexplained weight gain, cold intolerance, and fatigue. Physical examination reveals bradycardia and dry, cool skin. The healthcare provider suspects hypothyroidism and plans to order diagnostic tests. Which of the following thyroid function tests is the most sensitive indicator of primary hypothyroidism?
A) Thyroid-stimulating hormone (TSH)
B) Total thyroxine (T4) C) Total triiodothyronine
(T3) D) Free thyroxine (FT4)
E) Free triiodothyronine (FT3)

Your Answer:
QBankProAcademy.com

Rationale: This question assesses the nurse practitioner’s understanding of diagnostic tests for hypothyroidism. In primary hypothyroidism, the pituitary gland responds to low thyroid hormone levels by increasing TSH secretion. Therefore, measuring TSH levels is the most sensitive indicator of primary hypothyroidism.

AANP Question 20: A 28-year-old male patient presents with symptoms of excessive thirst, frequent urination, and unexplained weight loss. The healthcare provider suspects diabetes mellitus and plans to order diagnostic tests. Which of the following diagnostic tests provides immediate information about the patient’s blood glucose level at the time of testing?
A) Fasting blood glucose test
B) Oral glucose tolerance test (OGTT)
C) Glycosylated hemoglobin (HbA1c) test
D) Random blood glucose test
E) C-peptide test

Your Answer:
QBankProAcademy.com

Rationale: This question evaluates the nurse practitioner’s knowledge of diagnostic tests for diabetes mellitus. The random blood glucose test provides immediate information about the patient’s blood glucose level at the time of testing, making it useful for diagnosing diabetes mellitus during acute presentations.

AANP Question 21: A 55-year-old male patient presents with complaints of excessive sweating, heat intolerance, and hand tremors. The healthcare provider suspects hyperthyroidism and plans to order thyroid function tests. Which of the following thyroid function tests directly measures the concentration of biologically active thyroid hormone in the bloodstream?
A) Thyroid-stimulating hormone (TSH)
B) Total thyroxine (T4)
C) Total triiodothyronine (T3)
D) Free thyroxine (FT4)
E) Free triiodothyronine (FT3)

Your Answer:
QBankProAcademy.com

Rationale: This question assesses the nurse practitioner’s knowledge of thyroid function tests. Free thyroxine (FT4) directly measures the concentration of biologically active, unbound thyroid hormone in the bloodstream, providing a more accurate assessment of thyroid function than total T4.

Infection in adults and Children

AANP Question 22: A 45-year-old female patient presents with a fever, productive cough, and increased sputum production for the past three days. She has a history of chronic obstructive pulmonary disease (COPD) and is a former smoker. On examination, you note decreased breath sounds on the right side of the chest. Chest X-ray reveals a consolidation in the right lower lobe. What is the most likely causative organism for this patient’s condition?
A) Staphylococcus aureus
B) Streptococcus pneumoniae
C) Mycoplasma pneumoniae
D) Klebsiella pneumoniae
E) Haemophilus influenzae

Your Answer:
QBankProAcademy.com

Rationale: This question assesses the APRN’s ability to identify the likely causative organism of a lower respiratory tract infection in a patient with risk factors such as COPD and smoking history. Klebsiella pneumoniae is a common pathogen associated with pneumonia in patients with these risk factors.

NCLEX Pediatric Questions, AANP, ANCC, NCLEX, questions and answers HESI Exit
Hospitalized infant in the intensive care unit

AANP Question 23: A 30-year-old male patient presents with a painful, swollen finger. He works as a chef and has a small cut on his finger that occurred while chopping vegetables a few days ago. On examination, you observe erythema and purulent discharge from the affected finger. Which of the following is the most appropriate initial management for this patient’s infection?
A) Empiric oral antibiotics
B) Immediate surgical debridement
C) Topical antiseptic ointment
D) Warm compress and elevation
E) No intervention required

Your Answer:
QBankProAcademy.com

Rationale: This question assesses the APRN’s knowledge of managing a localized hand infection. In cases of an abscess or cellulitis with purulent discharge, immediate surgical debridement is often necessary to prevent further spread of infection.

AANP Question 24: A 60-year-old male patient with a history of type 2 diabetes mellitus presents with a deep foot ulcer that is foul-smelling and discharging pus. The ulcer has been present for several weeks and is not healing despite wound care interventions. On examination, you notice surrounding cellulitis and pitting edema of the lower extremity. Which of the following microorganisms is most commonly associated with diabetic foot ulcers of this nature?
A) Escherichia coli
B) Pseudomonas aeruginosa
C) Staphylococcus aureus
D) Clostridium perfringens
E) Streptococcus pyogenes

Your Answer:
QBankProAcademy.com

Rationale: This question evaluates the APRN’s understanding of the microbial pathogens commonly associated with diabetic foot ulcers. Staphylococcus aureus is a frequent causative organism in diabetic foot infections, especially in cases with cellulitis and non-healing ulcers. Understanding the microbiology helps guide appropriate antibiotic therapy.

AANP Question 25: A 35-year-old female patient presents to the clinic with complaints of a sore throat, fever, and cervical lymphadenopathy. On examination, you note tonsillar exudates and swollen tonsils. Her rapid streptococcal antigen test comes back positive. What is the most appropriate treatment for this patient’s infection?
A) Amoxicillin
B) Doxycycline
C) Azithromycin
D) Ciprofloxacin
E) Acyclovir

Your Answer:
QBankProAcademy.com

Rationale: This question assesses the APRN’s knowledge of the appropriate antibiotic therapy for a patient with a positive rapid streptococcal antigen test, which indicates a Group A Streptococcus (Streptococcus pyogenes) infection. Amoxicillin is the first-line treatment for streptococcal pharyngitis.

AANP, ANCC, NCLEX, questions and answers HESI Exit, Acute Gout
Acute Gout

AANP Question 26: A 50-year-old male patient presents with a painful, red, and swollen right knee joint. He has a history of gout and reports experiencing a recent gout flare in the same knee. On examination, the joint is warm to touch, and there is limited range of motion due to pain. Which of the following organisms is most likely responsible for this septic arthritis in a patient with a history of gout?
A) Staphylococcus aureus
B) Escherichia coli
C) Streptococcus pneumoniae
D) Neisseria gonorrhoeae
E) Mycoplasma pneumoniae

Your Answer:
QBankProAcademy.com

Rationale: This question evaluates the APRN’s understanding of septic arthritis and its association with specific pathogens. Staphylococcus aureus is a common causative organism in septic arthritis, especially in patients with risk factors such as gout.

AANP Question 27: A 28-year-old sexually active female presents with lower abdominal pain, dysuria, and an increase in vaginal discharge. On pelvic examination, you observe cervical motion tenderness and purulent cervical discharge. She mentions having multiple sexual partners and inconsistent condom use. What is the most likely causative organism for this patient’s infection?
A) Chlamydia trachomatis
B) Neisseria gonorrhoeae
C) Escherichia coli
D) Treponema pallidum
E) Candida albicans

Your Answer:
QBankProAcademy.com

Rationale: This question assesses the APRN’s ability to diagnose and identify the likely causative organism in a patient with symptoms suggestive of pelvic inflammatory disease (PID). Neisseria gonorrhoeae is a common pathogen associated with PID in sexually active individuals.

AANP Question 28: A 45-year-old male patient with a history of diabetes presents with a foot wound that is red, swollen, and draining purulent material. On examination, you note cellulitis surrounding the wound. Which of the following microorganisms is most commonly responsible for cellulitis in diabetic foot ulcers?
A) Escherichia coli
B) Staphylococcus aureus
C) Clostridium perfringens
D) Mycoplasma pneumoniae
E) Pseudomonas aeruginosa

Your Answer:
QBankProAcademy.com

Rationale: This question tests the APRN’s knowledge of the typical pathogens associated with diabetic foot ulcers. Staphylococcus aureus is one of the most common causative organisms in cellulitis related to diabetic foot wounds, especially in patients with diabetes.

AANP Question 29: A 30-year-old female patient presents with a high fever, chills, and productive cough with rust-colored sputum. On examination, you note dullness to percussion and decreased breath sounds over the right lower lung field. Chest X-ray reveals a lobar consolidation. What is the most likely causative organism for this patient’s condition?
A) Haemophilus influenzae
B) Klebsiella pneumoniae
C) Streptococcus pneumoniae
D) Mycobacterium tuberculosis
E) Pseudomonas aeruginosa

Your Answer:
QBankProAcademy.com

Rationale: This question assesses the APRN’s ability to identify the likely causative organism of a lobar consolidation in a patient with clinical features suggestive of community-acquired pneumonia. Streptococcus pneumoniae is a common pathogen associated with this presentation.

AANP Question 30: A 55-year-old male patient presents with a history of recurrent urinary tract infections (UTIs). He reports frequent urination, urgency, and dysuria. Urinalysis reveals pyuria and hematuria. What is the most likely causative organism for this patient’s recurrent UTIs?
A) Escherichia coli
B) Staphylococcus aureus
C) Candida albicans
D) Mycoplasma genitalium
E) Treponema pallidum

Your Answer:
QBankProAcademy.com

Rationale: This question evaluates the APRN’s understanding of common pathogens causing recurrent UTIs. Escherichia coli is the most common causative organism in uncomplicated UTIs and recurrent UTIs in adults, making it the likely culprit in this case.

AANP Question 31: A 32-year-old pregnant patient presents with a confirmed urinary tract infection (UTI) with no signs of systemic illness. She is in her second trimester of pregnancy. Which of the following antibiotics is the most appropriate choice for treating her UTI due to its safety profile during pregnancy?
A) Ciprofloxacin
B) Trimethoprim-sulfamethoxazole (TMP-SMX)
C) Nitrofurantoin
D) Doxycycline
E) Amoxicillin-clavulanate

Your Answer:
QBankProAcademy.com

Rationale: This question assesses the APRN’s knowledge of appropriate antibiotic choices for treating UTIs in pregnant patients. Nitrofurantoin is often considered the first-line treatment for UTIs during pregnancy due to its safety profile and effectiveness.

AANP Question 32: A 55-year-old male patient presents with community-acquired pneumonia. He has a history of chronic kidney disease and is currently on hemodialysis. Which antibiotic is most suitable for the initial empiric treatment of his pneumonia, taking into consideration his renal impairment?
A) Levofloxacin
B) Ceftriaxone
C) Azithromycin
D) Amoxicillin-clavulanate
E) Vancomycin

Your Answer:
QBankProAcademy.com

Rationale: This question tests the APRN’s ability to select an appropriate antibiotic for a patient with pneumonia and renal impairment. Ceftriaxone is often chosen for empiric treatment due to its renal excretion pathway, making it suitable for patients with kidney disease.

AANP Question 33: A 42-year-old female patient presents with a severe skin infection with signs of systemic toxicity, including fever, hypotension, and altered mental status. On examination, you observe extensive cellulitis and a rapidly spreading necrotic lesion. Which of the following antibiotics should be administered immediately for empiric broad-spectrum coverage in this critically ill patient?
A) Cefazolin
B) Clindamycin
C) Ciprofloxacin
D) Vancomycin
E) Trimethoprim-sulfamethoxazole (TMP-SMX)

Your Answer:
QBankProAcademy.com

Rationale: This question evaluates the APRN’s ability to recognize the need for immediate broad-spectrum antibiotic therapy in a critically ill patient with a severe skin infection suggestive of necrotizing fasciitis. Vancomycin provides coverage against methicillin-resistant Staphylococcus aureus (MRSA), a common pathogen in such cases.

AANP Question 34: A 28-year-old female patient presents with complaints of a burning sensation during urination, increased urinary frequency, and cloudy, foul-smelling urine. She denies fever or flank pain. On examination, there is mild suprapubic tenderness. Which of the following is the most likely diagnosis for this patient’s symptoms?
A) Acute pyelonephritis
B) Urinary tract infection (UTI)
C) Interstitial cystitis
D) Urolithiasis
E) Pelvic inflammatory disease (PID)

Your Answer:
QBankProAcademy.com

Rationale: This question evaluates the APRN’s ability to recognize common symptoms and clinical findings associated with a urinary tract infection (UTI). The patient’s complaints of dysuria, increased urinary frequency, and foul-smelling urine are indicative of a UTI.

AANP Question 35: A 45-year-old male patient presents with a gradual onset of fever, productive cough, and pleuritic chest pain. On auscultation, you hear coarse crackles over the right lower lung field. Chest X-ray reveals a patchy infiltrate in the right lower lobe. Which of the following conditions is most likely responsible for this patient’s presentation?
A) Acute bronchitis
B) Pulmonary embolism
C) Community-acquired pneumonia
D) Pleurisy
E) Asthma exacerbation

Your Answer:
QBankProAcademy.com

Rationale: This question assesses the APRN’s ability to differentiate between common respiratory conditions based on clinical presentation and radiographic findings. The patient’s fever, productive cough, pleuritic chest pain, and lung examination findings are suggestive of community-acquired pneumonia.

AANP Question 36: A 60-year-old male patient presents with complaints of abdominal pain, diarrhea, and unintentional weight loss over the past several weeks. He mentions recent international travel to South Asia. Stool studies reveal the presence of cysts with multiple nuclei. What is the most likely causative organism for this patient’s gastrointestinal symptoms?
A) Entamoeba histolytica
B) Giardia lamblia
C) Cryptosporidium parvum
D) Clostridium difficile
E) Salmonella enterica

Your Answer:
QBankProAcademy.com

Rationale: This question evaluates the APRN’s ability to identify the likely causative organism in a patient with gastrointestinal symptoms following international travel. Entamoeba histolytica is a protozoan parasite commonly associated with abdominal pain, diarrhea, and weight loss in travelers returning from endemic regions.

AANP Question 37: A 22-year-old male patient presents with a sore throat, fever, and enlarged cervical lymph nodes. On examination, you observe tonsillar exudates and anterior cervical lymphadenopathy. Which of the following tests is most appropriate to confirm the diagnosis in this patient?
A) Complete blood count (CBC)
B) Rapid streptococcal antigen test (Strep test)
C) Monospot test for Epstein-Barr virus (EBV)
D) Chest X-ray
E) Throat culture

Your Answer:
QBankProAcademy.com

Correct Answer: B) Rapid streptococcal antigen test (Strep test)

Rationale: This question assesses the APRN’s ability to select the most appropriate diagnostic test for a patient with clinical symptoms suggestive of streptococcal pharyngitis. The rapid streptococcal antigen test (Strep test) is a quick and reliable way to confirm the presence of Group A Streptococcus, a common cause of sore throat.

AANP Question 38: A 35-year-old female patient presents with lower abdominal pain, fever, and purulent vaginal discharge. On pelvic examination, you note cervical motion tenderness and adnexal tenderness. Which of the following tests is essential for confirming the diagnosis and guiding appropriate treatment in this patient?
A) Complete blood count (CBC)
B) Urinalysis
C) Transvaginal ultrasound
D) Wet mount microscopy of vaginal discharge
E) Throat culture

Your Answer:
QBankProAcademy.com

Rationale: This question evaluates the APRN’s knowledge of diagnostic methods for pelvic inflammatory disease (PID). Wet mount microscopy of vaginal discharge can help identify the presence of Trichomonas vaginalis and clue cells, which are indicative of PID.

AANP Question 39: A 50-year-old male patient presents with fever, night sweats, and unintentional weight loss. He has a history of HIV infection with a CD4 count of 150 cells/mm³. On physical examination, you observe enlarged cervical and axillary lymph nodes. Which of the following tests is most appropriate for diagnosing the likely cause of his symptoms?
A) Tuberculin skin test (TST)
B) HIV viral load
C) Complete blood count (CBC)
D) Chest X-ray
E) Rapid plasma reagin (RPR) test for syphilis

Your Answer:
QBankProAcademy.com

Rationale: This question assesses the APRN’s ability to identify the most appropriate diagnostic test for a patient with symptoms suggestive of HIV-associated tuberculosis (TB). Chest X-ray is a crucial initial step in diagnosing pulmonary TB in individuals with HIV, especially when presenting with fever, night sweats, and weight loss.

AANP Question 40: A 28-year-old sexually active female patient presents with lower abdominal pain, vaginal discharge, and dysuria. On pelvic examination, you note cervical motion tenderness and purulent cervical discharge. Which of the following diagnostic tests is most appropriate to confirm the likely diagnosis in this patient?
A) Complete blood count (CBC)
B) Transvaginal ultrasound
C) Throat culture
D) Nucleic acid amplification test (NAAT) for Chlamydia trachomatis and Neisseria gonorrhoeae
E) Tuberculin skin test (TST)

Your Answer:
QBankProAcademy.com

Rationale: This question assesses the APRN’s knowledge of appropriate diagnostic tests for suspected sexually transmitted infections (STIs). A NAAT for Chlamydia trachomatis and Neisseria gonorrhoeae is a highly sensitive and specific test for confirming the diagnosis of these common STIs in symptomatic patients.

AANP Question 41: A 65-year-old male patient with a history of chronic obstructive pulmonary disease (COPD) presents with increased cough, sputum production, and fever. On examination, you note decreased breath sounds and dullness to percussion over the right lower lung field. Which of the following diagnostic tests is most appropriate to confirm the diagnosis in this patient?
A) Complete blood count (CBC)
B) Chest X-ray
C) Throat culture
D) Stool culture
E) Skin biopsy

Your Answer:
QBankProAcademy.com

Rationale: This question evaluates the APRN’s ability to select the most appropriate diagnostic test for a patient with clinical symptoms suggestive of a lower respiratory tract infection. Chest X-ray is a key diagnostic tool for identifying the presence of a lobar consolidation, which could indicate pneumonia in a patient with COPD.

AANP Question 42: A 40-year-old male patient presents with a painful, swollen finger. He works in construction and reports sustaining a small puncture wound from a nail several days ago. On examination, you observe erythema, warmth, and purulent discharge from the affected finger. Which of the following diagnostic tests is most appropriate to confirm the diagnosis in this patient?
A) Complete blood count (CBC)
B) Joint aspiration
C) Throat culture
D) Wound culture
E) Electrocardiogram (ECG)

Your Answer:
QBankProAcademy.com

Rationale: This question assesses the APRN’s understanding of the diagnostic approach for localized infections. In cases of a purulent wound infection, obtaining a wound culture is crucial to identify the causative organism and guide appropriate antibiotic therapy.

AANP Question 43: A 30-year-old female patient presents with fever, headache, and a maculopapular rash that started on her face and has spread to her trunk and extremities. She recently returned from a trip to a tropical region. On examination, you notice conjunctival injection and small white lesions on her buccal mucosa. What diagnostic test is most appropriate to confirm the suspected diagnosis in this patient?
A) Stool culture
B) Blood culture
C) Skin biopsy
D) Urinalysis
E) Throat culture

Your Answer:
QBankProAcademy.com

Rationale: This question assesses the APRN’s knowledge of diagnostic procedures for infectious diseases with distinctive clinical presentations. The patient’s symptoms, including fever, rash, conjunctival injection, and buccal mucosa lesions, are suggestive of measles. A skin biopsy can confirm the diagnosis by identifying characteristic viral inclusion bodies.

AANP Question 44: A 45-year-old male patient presents with high fever, severe headache, joint pain, and a rash that began on his wrists and ankles and spread to involve his trunk. He recently returned from a camping trip where he recalls being bitten by multiple ticks. What diagnostic test is most appropriate for confirming the likely diagnosis in this patient?
A) Stool culture
B) Blood culture
C) Polymerase chain reaction (PCR) for Lyme disease
D) Chest X-ray
E) Throat culture

Your Answer:
QBankProAcademy.com

Rationale: This question evaluates the APRN’s understanding of diagnostic methods for tick-borne infections. The patient’s symptoms, history of tick exposure, and rash distribution are suggestive of Lyme disease. PCR testing is useful for detecting Borrelia burgdorferi DNA in the early stages of the disease.

AANP Question 45: A 60-year-old male patient presents with cough, fever, night sweats, and unintentional weight loss. He has a history of diabetes mellitus and a remote history of tuberculosis (TB) exposure. On physical examination, you note decreased breath sounds and dullness to percussion over the right upper lung field. What diagnostic test is most appropriate for confirming the likely diagnosis in this patient?
A) Stool culture
B) Blood culture
C) Sputum acid-fast bacilli (AFB) smear and culture
D) Urinalysis
E) Throat culture

Your Answer:
QBankProAcademy.com

Rationale: This question assesses the APRN’s ability to identify the most appropriate diagnostic test for a patient with clinical symptoms suggestive of pulmonary tuberculosis (TB). The patient’s cough, fever, night sweats, weight loss, and chest examination findings are concerning for TB, and sputum AFB smear and culture are key tests for diagnosis.

AANP Question 46: A 35-year-old male patient with a history of intravenous drug use presents with fever, chills, and a tender, erythematous lesion with surrounding warmth on his forearm. On examination, you notice a fluctuant abscess at the site. Which of the following is the most likely causative organism for this patient’s skin infection?
A) Escherichia coli
B) Streptococcus pyogenes
C) Pseudomonas aeruginosa
D) Clostridium perfringens
E) Staphylococcus aureus

Your Answer:
QBankProAcademy.com

Rationale: This question evaluates the APRN’s ability to identify the likely causative organism in a patient with a skin abscess related to intravenous drug use. Staphylococcus aureus, including methicillin-resistant Staphylococcus aureus (MRSA), is a common pathogen in such cases.

AANP Question 47: A 28-year-old sexually active female presents with lower abdominal pain, vaginal discharge, and dysuria. On pelvic examination, you note cervical motion tenderness and adnexal tenderness. Which of the following organisms is most commonly associated with the condition in this patient?
A) Escherichia coli
B) Neisseria gonorrhoeae
C) Candida albicans
D) Treponema pallidum
E) Mycoplasma pneumoniae

Your Answer:
QBankProAcademy.com

Rationale: This question assesses the APRN’s understanding of common pathogens causing pelvic inflammatory disease (PID) in sexually active individuals. Neisseria gonorrhoeae is a frequent causative organism in PID, often presenting with lower abdominal pain, vaginal discharge, and pelvic tenderness.

AANP Question 48: A 60-year-old male patient presents with cough, fever, night sweats, and unintentional weight loss. He has a history of chronic obstructive pulmonary disease (COPD) and a remote history of tuberculosis (TB) exposure. On examination, you note decreased breath sounds and dullness to percussion over the right upper lung field. Which of the following is the most likely causative organism for this patient’s condition?
A) Haemophilus influenzae
B) Streptococcus pneumoniae
C) Mycobacterium tuberculosis
D) Pseudomonas aeruginosa
E) Klebsiella pneumoniae

Your Answer:
QBankProAcademy.com

Rationale: This question assesses the APRN’s ability to recognize the likely causative organism in a patient with clinical symptoms suggestive of pulmonary tuberculosis (TB). The patient’s cough, fever, night sweats, weight loss, and chest examination findings are classic features of TB, which is caused by Mycobacterium tuberculosis.

AANP Question 49: A 3-year-old child presents with a high fever, irritability, and a stiff neck. On physical examination, you note photophobia and a positive Brudzinski sign. The child’s fontanelle is bulging. What is the most likely causative organism for this child’s condition?
A) Haemophilus influenzae type b (Hib)
B) Streptococcus pneumoniae
C) Neisseria meningitidis
D) Enterovirus
E) Staphylococcus aureus

Your Answer:
QBankProAcademy.com

Rationale: This question assesses the APRN’s knowledge of pediatric infections, specifically pediatric bacterial meningitis. The clinical presentation, including fever, irritability, photophobia, positive Brudzinski sign, and bulging fontanelle, is highly suggestive of bacterial meningitis, and Hib is a common causative organism in young children.

AANP Question 50: A 5-year-old child presents with a sore throat, fever, and swollen cervical lymph nodes. On examination, you note tonsillar exudates. The child’s parent reports recent exposure to a family member with a similar illness. What is the most likely causative organism for this child’s condition?
A) Streptococcus pneumoniae
B) Epstein-Barr virus (EBV)
C) Streptococcus pyogenes (Group A Streptococcus)
D) Human herpesvirus 6 (HHV-6)
E) Rhinovirus

Your Answer:
QBankProAcademy.com

Rationale: This question assesses the APRN’s ability to diagnose a common pediatric infection, specifically streptococcal pharyngitis (strep throat). The symptoms of sore throat, fever, tonsillar exudates, and recent exposure to a family member with a similar illness are indicative of a streptococcal infection.

AANP Question 51: A 2-year-old child presents with diarrhea, abdominal cramps, and vomiting. The child attends daycare, and several other children in the same facility have similar symptoms. What is the most likely causative organism for this child’s gastrointestinal illness?

A) Shigella spp. B) Escherichia coli O157:H7 C) Rotavirus D) Norovirus E) Salmonella spp.

Your Answer:
QBankProAcademy.com

Correct Answer: D) Norovirus

Rationale: This question evaluates the APRN’s ability to identify the likely causative organism in a child with symptoms of gastroenteritis. Norovirus is a common cause of outbreaks of gastrointestinal illness in daycare settings and is characterized by symptoms such as diarrhea, abdominal cramps, and vomiting.

AANP Question 52: A 4-year-old child presents with a high fever, cough, and rapid breathing. On physical examination, you note inspiratory stridor, nasal flaring, and intercostal retractions. The child appears anxious and irritable. What is the most likely causative organism for this child’s condition?

A) Streptococcus pneumoniae B) Respiratory syncytial virus (RSV) C) Haemophilus influenzae type b (Hib) D) Mycoplasma pneumoniae E) Parainfluenza virus

Your Answer:
QBankProAcademy.com

Correct Answer: B) Respiratory syncytial virus (RSV)

Rationale: This question evaluates the APRN’s ability to recognize the clinical presentation of bronchiolitis in a young child and identify the most common causative organism. The symptoms, including high fever, cough, inspiratory stridor, and intercostal retractions, are typical of severe RSV bronchiolitis.

AANP Question 53: A 6-year-old child presents with a pruritic rash that started on the face and has spread to involve the trunk and extremities. On examination, you notice multiple vesicles on an erythematous base, with some lesions in different stages of development. The child’s parent reports that several classmates have similar rashes. What is the most likely causative organism for this child’s condition?

A) Streptococcus pyogenes (Group A Streptococcus) B) Herpes simplex virus (HSV) C) Varicella-zoster virus (VZV) D) Enterovirus E) Measles virus

Your Answer:
QBankProAcademy.com

Correct Answer: C) Varicella-zoster virus (VZV)

Rationale: This question assesses the APRN’s knowledge of pediatric viral infections and their characteristic rashes. The description of pruritic vesicles on an erythematous base in different stages of development is classic for varicella (chickenpox), caused by VZV.

AANP Question 54: A 10-year-old child presents with a sudden onset of high fever, headache, photophobia, and a stiff neck. On physical examination, you note a positive Brudzinski sign and Kernig sign. The child’s parent reports that there have been cases of meningitis in the school recently. What is the most likely causative organism for this child’s condition?

A) Neisseria meningitidis B) Streptococcus pneumoniae C) Haemophilus influenzae type b (Hib) D) Enterovirus E) Epstein-Barr virus (EBV)

Your Answer:
QBankProAcademy.com

Correct Answer: A) Neisseria meningitidis

Rationale: This question evaluates the APRN’s ability to recognize the clinical presentation of acute bacterial meningitis in a child and identify the most common causative organism. The sudden onset of high fever, headache, photophobia, stiff neck, and positive Brudzinski and Kernig signs are indicative of acute bacterial meningitis, with Neisseria meningitidis being a common pathogen in school outbreaks.

AANP Question 55: A 2-month-old infant presents with fever, irritability, poor feeding, and a bulging fontanelle. On physical examination, you note nuchal rigidity and photophobia. What is the most likely causative organism for this infant’s condition?

A) Group B Streptococcus (GBS) B) Escherichia coli C) Haemophilus influenzae type b (Hib) D) Listeria monocytogenes E) Neisseria meningitidis

Your Answer:
QBankProAcademy.com

Correct Answer: A) Group B Streptococcus (GBS)

Rationale: This question assesses the APRN’s knowledge of neonatal bacterial meningitis, a serious infection in infants. The clinical presentation, including fever, irritability, bulging fontanelle, nuchal rigidity, and photophobia, is suggestive of bacterial meningitis, with GBS being a common causative organism in neonates.

AANP Question 56: A 3-week-old infant presents with nasal congestion, cough, and noisy breathing. The child’s parent reports difficulty with feeding due to breathing problems. On examination, you observe inspiratory stridor, chest retractions, and nasal flaring. What is the most likely causative organism for this infant’s condition?

A) Respiratory syncytial virus (RSV) B) Streptococcus pneumoniae C) Haemophilus influenzae type b (Hib) D) Bordetella pertussis E) Influenza virus

Your Answer:
QBankProAcademy.com

Correct Answer: D) Bordetella pertussis

Rationale: This question evaluates the APRN’s knowledge of pertussis (whooping cough) in infants. The symptoms, including nasal congestion, cough with inspiratory stridor, and feeding difficulties, are suggestive of pertussis, which is caused by Bordetella pertussis.

AANP Question 57: A 6-month-old infant presents with fever, irritability, vomiting, and diarrhea. The child’s parent reports several loose, foul-smelling stools per day. On examination, you notice abdominal distension and tenderness. What is the most likely causative organism for this infant’s condition?

A) Rotavirus B) Escherichia coli C) Norovirus D) Salmonella enterica E) Clostridium difficile

Your Answer:
QBankProAcademy.com

Correct Answer: E) Clostridium difficile

Rationale: This question assesses the APRN’s knowledge of infantile gastroenteritis and the potential involvement of Clostridium difficile. The infant’s clinical presentation, including fever, vomiting, diarrhea, abdominal distension, and tenderness, may be indicative of C. difficile-associated diarrhea, which can occur in infants.

AANP Question 58: A 2-week-old infant presents with fever, poor feeding, and irritability. On examination, you notice a bulging fontanelle and nuchal rigidity. What is the most likely causative organism for this infant’s condition?

A) Streptococcus pneumoniae B) Group B Streptococcus (GBS) C) Escherichia coli D) Listeria monocytogenes E) Haemophilus influenzae type b (Hib)

Your Answer:
QBankProAcademy.com

Correct Answer: B) Group B Streptococcus (GBS)

Rationale: This question assesses the APRN’s knowledge of neonatal bacterial meningitis, a serious infection in infants. The clinical presentation, including fever, irritability, bulging fontanelle, and nuchal rigidity, is suggestive of bacterial meningitis, with GBS being a common causative organism in neonates.

AANP Question 59: A 4-week-old infant presents with a hoarse cry, poor feeding, and difficulty breathing. On examination, you observe inspiratory stridor, nasal flaring, and retractions. What is the most likely causative organism for this infant’s condition?

A) Respiratory syncytial virus (RSV) B) Haemophilus influenzae type b (Hib) C) Bordetella pertussis D) Influenza virus E) Parainfluenza virus

Your Answer:
QBankProAcademy.com

Correct Answer: C) Bordetella pertussis

Rationale: This question evaluates the APRN’s knowledge of pertussis (whooping cough) in infants. The symptoms, including hoarse cry, inspiratory stridor, nasal flaring, and difficulty breathing, are suggestive of pertussis, which is caused by Bordetella pertussis.

AANP Question 60: A 2-month-old infant presents with fever, vomiting, and diarrhea. The child’s parent reports several watery stools per day. On examination, you notice abdominal distension and tenderness. What is the most likely causative organism for this infant’s condition?

A) Rotavirus B) Escherichia coli C) Norovirus D) Clostridium difficile E) Salmonella enterica

Your Answer:
QBankProAcademy.com

Correct Answer: A) Rotavirus

Rationale: This question assesses the APRN’s knowledge of infantile gastroenteritis and the potential involvement of rotavirus. The infant’s clinical presentation, including fever, vomiting, watery diarrhea, abdominal distension, and tenderness, is characteristic of rotavirus infection, a common cause of gastroenteritis in infants.

AANP Question 61: A 1-month-old infant presents with fever, irritability, and difficulty in feeding. On examination, you observe a bulging fontanelle and a positive Brudzinski sign. What is the most likely causative organism for this infant’s condition?

A) Streptococcus pneumoniae B) Group B Streptococcus (GBS) C) Listeria monocytogenes D) Haemophilus influenzae type b (Hib) E) Escherichia coli

Your Answer:
QBankProAcademy.com

Correct Answer: D) Haemophilus influenzae type b (Hib)

Rationale: This question evaluates the APRN’s knowledge of neonatal bacterial meningitis, a serious infection in infants. The clinical presentation, including fever, irritability, bulging fontanelle, and positive Brudzinski sign, is suggestive of bacterial meningitis, with Hib being a common causative organism in neonates.

AANP Question 62: A 6-week-old infant presents with a hoarse cry, cough, and difficulty breathing. On examination, you notice inspiratory stridor, nasal flaring, and chest retractions. What is the most likely causative organism for this infant’s condition?

A) Respiratory syncytial virus (RSV) B) Haemophilus influenzae type b (Hib) C) Bordetella pertussis D) Influenza virus E) Parainfluenza virus

Your Answer:
QBankProAcademy.com

Correct Answer: C) Bordetella pertussis

Rationale: This question assesses the APRN’s knowledge of pertussis (whooping cough) in infants. The symptoms, including hoarse cry, inspiratory stridor, nasal flaring, and difficulty breathing, are suggestive of pertussis, which is caused by Bordetella pertussis.

AANP Question 63: A 2-month-old infant presents with fever, vomiting, and watery diarrhea. The child’s parent reports several episodes of diarrhea per day. On examination, you notice mild abdominal distension. What is the most likely causative organism for this infant’s condition?

A) Rotavirus B) Escherichia coli C) Norovirus D) Clostridium difficile E) Salmonella enterica

Your Answer:
QBankProAcademy.com

Correct Answer: A) Rotavirus

Rationale: This question assesses the APRN’s knowledge of infantile gastroenteritis and the potential involvement of rotavirus. The infant’s clinical presentation, including fever, vomiting, watery diarrhea, and mild abdominal distension, is characteristic of rotavirus infection, a common cause of gastroenteritis in infants.

AANP Question 64: A 55-year-old patient with a history of hypertension presents to the clinic for a follow-up visit. The patient reports compliance with prescribed antihypertensive medications but mentions experiencing occasional dizziness. On examination, the blood pressure is 150/90 mm Hg. Which of the following actions should the APRN take first?

Multiple Choice Answers: A) Increase the current antihypertensive medication dosage. B) Order a 24-hour ambulatory blood pressure monitoring (ABPM) test. C) Refer the patient to a cardiologist for further evaluation. D) Prescribe a diuretic as an additional antihypertensive. E) Educate the patient on lifestyle modifications.

Correct Answer: B) Order a 24-hour ambulatory blood pressure monitoring (ABPM) test.

Rationale: This question assesses the APRN’s ability to identify an appropriate action when managing a hypertensive patient with symptoms of dizziness. Ordering a 24-hour ABPM test is recommended to assess blood pressure variability and rule out white coat hypertension, which can help guide treatment decisions and avoid unnecessary medication adjustments.

AANP Question 65: A 45-year-old patient with newly diagnosed hypertension presents to the APRN for initial assessment. The patient has no other significant medical history and is not currently taking any medications. The initial blood pressure reading is 160/100 mm Hg. What is the most appropriate first-line pharmacological treatment option for this patient?

Multiple Choice Answers: A) ACE inhibitor (e.g., lisinopril) B) Beta-blocker (e.g., metoprolol) C) Calcium channel blocker (e.g., amlodipine) D) Thiazide diuretic (e.g., hydrochlorothiazide) E) Alpha-blocker (e.g., doxazosin)

Correct Answer: D) Thiazide diuretic (e.g., hydrochlorothiazide)

Rationale: This question assesses the APRN’s knowledge of first-line pharmacological options for the treatment of hypertension in a newly diagnosed patient. Thiazide diuretics are often recommended as initial therapy in patients with uncomplicated hypertension, as they are effective and well-tolerated.

AANP Question 66: A 65-year-old patient with a history of hypertension and diabetes mellitus presents to the clinic for a routine follow-up. The patient is currently taking an ACE inhibitor for blood pressure control and metformin for diabetes management. Laboratory results show a serum potassium level of 5.8 mEq/L (normal range: 3.5-5.0 mEq/L). What should the APRN do next?

Multiple Choice Answers: A) Discontinue the ACE inhibitor and start an ARB. B) Decrease the metformin dosage. C) Prescribe a potassium-sparing diuretic. D) Recommend dietary potassium restriction. E) Continue current medications and monitor potassium levels.

Correct Answer: E) Continue current medications and monitor potassium levels.

Rationale: This question assesses the APRN’s ability to manage a patient with hypertension and elevated serum potassium levels. In this scenario, it is essential to continue the current medications (ACE inhibitor and metformin) and monitor potassium levels. A single elevated potassium level may not necessitate medication changes, but ongoing monitoring is crucial to ensure patient safety.

AANP Question 67: A 62-year-old patient with a history of hypertension presents with a blood pressure reading of 160/95 mm Hg. The patient has been on a stable dose of an ACE inhibitor for the last year. During the assessment, the patient mentions a persistent dry cough. Which of the following actions should the APRN consider first?

Multiple Choice Answers: A) Increase the dosage of the ACE inhibitor. B) Switch to an angiotensin receptor blocker (ARB). C) Prescribe a calcium channel blocker. D) Add a beta-blocker to the current therapy. E) Recommend lifestyle modifications.

Correct Answer: B) Switch to an angiotensin receptor blocker (ARB).

Rationale: The persistent dry cough is a common side effect of ACE inhibitors. In this scenario, switching the patient to an ARB is an appropriate course of action to manage hypertension while avoiding the side effect of cough associated with ACE inhibitors.

AANP Question 68: A 50-year-old patient with newly diagnosed hypertension comes to the clinic. The patient has no other significant medical history. The initial blood pressure reading is 170/100 mm Hg. Which of the following lifestyle modifications should the APRN recommend as the initial intervention?

Multiple Choice Answers: A) Prescribe an antihypertensive medication. B) Advise daily consumption of a high-sodium diet. C) Encourage regular aerobic exercise. D) Recommend smoking cessation. E) Suggest increasing caffeine intake.

Correct Answer: C) Encourage regular aerobic exercise.

Rationale: Lifestyle modifications are often the initial intervention for patients with newly diagnosed hypertension, especially in the absence of other significant medical conditions. Regular aerobic exercise is known to have a positive impact on blood pressure control and overall cardiovascular health.

AANP Question 69: A 58-year-old patient with a history of hypertension presents with a blood pressure reading of 145/88 mm Hg. The patient is currently taking a calcium channel blocker and reports adherence to the medication regimen. However, blood pressure remains elevated. What is the most appropriate next step for the APRN?

Multiple Choice Answers: A) Increase the dosage of the calcium channel blocker. B) Switch to an ACE inhibitor. C) Prescribe a thiazide diuretic as an add-on therapy. D) Recommend stress reduction techniques. E) Discontinue the medication and monitor blood pressure.

Correct Answer: C) Prescribe a thiazide diuretic as an add-on therapy.

Rationale: In patients with uncontrolled hypertension on monotherapy, the addition of a thiazide diuretic is a common next step to enhance blood pressure control. This combination therapy is often more effective than increasing the dosage of a single medication and is supported by clinical guidelines.

AANP Question 70: A 60-year-old patient with a history of hypertension presents to the primary care clinic for a routine follow-up. The patient has been taking an ACE inhibitor for blood pressure management and reports good adherence to the medication. During the assessment, the patient mentions experiencing swelling in the ankles and a persistent dry cough. The blood pressure reading is 148/90 mm Hg. What should the APRN consider as the next course of action?

Multiple Choice Answers: A) Increase the dosage of the ACE inhibitor. B) Switch to an angiotensin receptor blocker (ARB). C) Add a calcium channel blocker to the current therapy. D) Recommend lifestyle modifications. E) Prescribe a beta-blocker as an additional antihypertensive.

Correct Answer: B) Switch to an angiotensin receptor blocker (ARB).

Rationale: The patient’s symptoms of ankle swelling and dry cough are common side effects associated with ACE inhibitors. Switching to an ARB is a suitable approach to manage hypertension while avoiding these adverse effects.

AANP Question 71: A 45-year-old patient with newly diagnosed hypertension presents to the APRN for evaluation. The patient has no other significant medical history and is not currently taking any medications. The initial blood pressure reading is 160/100 mm Hg. What is the most appropriate initial intervention for this patient?

Multiple Choice Answers: A) Prescribe a thiazide diuretic as the first-line medication. B) Recommend lifestyle modifications and reassess in 3 months. C) Start antihypertensive therapy with an ACE inhibitor. D) Order a 24-hour ambulatory blood pressure monitoring (ABPM) test. E) Suggest immediate hospitalization for blood pressure control.

Correct Answer: B) Recommend lifestyle modifications and reassess in 3 months.

Rationale: Lifestyle modifications are often the initial intervention for newly diagnosed hypertension, especially when there are no significant comorbidities. The patient should be advised on diet, exercise, and other lifestyle changes, and their blood pressure should be reevaluated after a period of lifestyle modification.

AANP Question 72: A 55-year-old patient with a history of hypertension presents with a blood pressure reading of 150/92 mm Hg. The patient is currently taking a beta-blocker for blood pressure control but continues to have elevated readings. What should the APRN consider as the next step in management?

Multiple Choice Answers: A) Increase the dosage of the beta-blocker. B) Add an ACE inhibitor to the current therapy. C) Switch to a calcium channel blocker. D) Recommend stress reduction techniques. E) Order a 24-hour ambulatory blood pressure monitoring (ABPM) test.

Correct Answer: C) Switch to a calcium channel blocker.

Rationale: If a patient’s blood pressure remains uncontrolled on monotherapy, it is common to consider switching to a different class of antihypertensive medication with a complementary mechanism of action. In this case, switching to a calcium channel blocker can be a suitable option to achieve better blood pressure control.

AANP Question 73: A 45-year-old patient with a history of hypertension presents to the clinic for a follow-up appointment. The patient reports taking antihypertensive medication regularly and adhering to a low-sodium diet. However, the blood pressure reading is consistently elevated at 160/95 mm Hg during multiple visits. What should the APRN do next?

Multiple Choice Answers: A) Increase the current antihypertensive medication dosage. B) Recommend dietary salt restriction and lifestyle modifications. C) Order a 24-hour ambulatory blood pressure monitoring (ABPM) test. D) Refer the patient to a cardiologist for further evaluation. E) Prescribe a diuretic as an additional antihypertensive.

Correct Answer: C) Order a 24-hour ambulatory blood pressure monitoring (ABPM) test.

Rationale: The elevated blood pressure readings in the clinic may be influenced by white coat hypertension or other factors. Ordering a 24-hour ABPM test can provide a more accurate assessment of the patient’s blood pressure patterns and guide treatment decisions.

AANP Question 74: A 55-year-old patient with a history of hypertension and type 2 diabetes presents for a routine check-up. The patient is currently taking an ACE inhibitor for blood pressure control. The latest laboratory results indicate a serum potassium level of 5.8 mEq/L (normal range: 3.5-5.0 mEq/L). What should the APRN do next?

Multiple Choice Answers: A) Increase the dosage of the ACE inhibitor. B) Add a calcium channel blocker to the current therapy. C) Discontinue the ACE inhibitor and prescribe an ARB. D) Recommend dietary potassium restriction. E) Continue current medications and monitor potassium levels.

Correct Answer: E) Continue current medications and monitor potassium levels.

Rationale: An elevated serum potassium level may be a side effect of ACE inhibitors, but it does not necessarily require discontinuation. In this case, it is essential to continue current medications and closely monitor potassium levels to ensure patient safety.

AANP Question 75: A 65-year-old patient with a history of hypertension presents for a routine follow-up. The patient has been taking a diuretic and an ACE inhibitor for blood pressure control. During the assessment, the patient mentions experiencing persistent dry mouth and increased thirst. The blood pressure reading is 138/86 mm Hg. What should the APRN consider as the next step in management?

Multiple Choice Answers: A) Increase the dosage of the diuretic. B) Prescribe an additional antihypertensive medication. C) Recommend a switch to an ARB. D) Assess for signs of hyperglycemia and diabetes. E) Advise the patient to increase fluid intake.

Correct Answer: D) Assess for signs of hyperglycemia and diabetes.

Rationale: Dry mouth and increased thirst could be symptoms of hyperglycemia, which may indicate the presence of diabetes. In this scenario, it is important to assess for signs of diabetes and consider appropriate diagnostic testing before making further changes to the hypertension treatment plan.

AANP Question 76: A 50-year-old patient presents to the primary care clinic for a routine check-up. The patient has a family history of hypertension and has not had their blood pressure checked in several years. On examination, the blood pressure reading is 150/95 mm Hg. The patient is asymptomatic and has no other significant medical history. What is the most appropriate initial action for the APRN?

Multiple Choice Answers: A) Initiate antihypertensive medication immediately. B) Recommend lifestyle modifications and recheck blood pressure in 1 month. C) Order a 24-hour ambulatory blood pressure monitoring (ABPM) test. D) Refer the patient to a cardiologist for further evaluation. E) Prescribe a thiazide diuretic as the first-line treatment.

Correct Answer: B) Recommend lifestyle modifications and recheck blood pressure in 1 month.

Rationale: In patients with newly diagnosed hypertension who are asymptomatic and have no significant comorbidities, initial lifestyle modifications are recommended. Rechecking blood pressure in a month allows for assessment of the effectiveness of lifestyle changes before considering medication.

AANP Question 77: A 60-year-old patient with a history of hypertension and diabetes mellitus presents for a follow-up appointment. The patient is currently taking an ACE inhibitor and metformin. The blood pressure reading is 152/92 mm Hg, and the glycated hemoglobin (HbA1c) level is 7.5% (normal range: 4.0-5.6%). What is the most appropriate adjustment to the patient’s treatment plan?

Multiple Choice Answers: A) Increase the dosage of the ACE inhibitor. B) Add a calcium channel blocker to the current therapy. C) Prescribe a thiazide diuretic as an add-on therapy. D) Refer the patient to a diabetes educator for dietary counseling. E) Recommend stress reduction techniques.

Correct Answer: C) Prescribe a thiazide diuretic as an add-on therapy.

Rationale: The patient’s blood pressure is not well controlled despite being on an ACE inhibitor. Adding a thiazide diuretic as an adjunctive therapy is a common strategy to achieve better blood pressure control, especially in patients with diabetes.

AANP Question 78: A 70-year-old patient with a history of hypertension presents to the APRN with a blood pressure reading of 160/100 mm Hg. The patient has been on a calcium channel blocker for several years and reports good adherence to the medication. During the visit, the patient mentions experiencing leg swelling and constipation. What should the APRN consider as the next step in management?

Multiple Choice Answers: A) Increase the dosage of the calcium channel blocker. B) Switch to an ACE inhibitor. C) Prescribe a thiazide diuretic as an add-on therapy. D) Evaluate for possible peripheral artery disease. E) Refer the patient to a gastroenterologist for constipation management.

Correct Answer: D) Evaluate for possible peripheral artery disease.

Rationale: Leg swelling and constipation could be signs of peripheral artery disease, especially in an older patient with hypertension. It is essential to assess for this condition before making changes to the antihypertensive medication regimen.

AANP Question 79: A 58-year-old patient with a history of hypertension presents to the primary care clinic for a routine follow-up. The patient has been taking an ACE inhibitor for blood pressure control and reports good adherence to the medication. The latest blood pressure reading is consistently elevated at 160/100 mm Hg. The patient has no other significant medical history or symptoms. What should the APRN do next?

Multiple Choice Answers: A) Increase the dosage of the ACE inhibitor. B) Prescribe a beta-blocker as an additional antihypertensive. C) Switch to an angiotensin receptor blocker (ARB). D) Recommend lifestyle modifications. E) Order a 24-hour ambulatory blood pressure monitoring (ABPM) test.

Correct Answer: E) Order a 24-hour ambulatory blood pressure monitoring (ABPM) test.

Rationale: In patients with persistent elevated blood pressure readings in the clinic, it is essential to rule out white coat hypertension and assess for blood pressure variability. Ordering a 24-hour ABPM test can provide a more accurate picture of the patient’s blood pressure patterns and guide treatment decisions.

AANP Question 80: A 52-year-old patient with newly diagnosed hypertension presents to the APRN for evaluation. The patient has no other significant medical history and is not currently taking any medications. The initial blood pressure reading is 170/100 mm Hg. What is the most appropriate initial intervention for this patient?

Multiple Choice Answers: A) Prescribe a thiazide diuretic as the first-line medication. B) Recommend lifestyle modifications and recheck blood pressure in 2 weeks. C) Start antihypertensive therapy with an ACE inhibitor. D) Order a renal artery ultrasound to assess for renovascular hypertension. E) Advise immediate hospitalization for blood pressure control.

Correct Answer: B) Recommend lifestyle modifications and recheck blood pressure in 2 weeks.

Rationale: Lifestyle modifications are often the initial intervention for newly diagnosed hypertension, especially when there are no significant comorbidities. Rechecking blood pressure after a period of lifestyle changes allows for assessment of their effectiveness before considering medication.

AANP Question 81: A 62-year-old patient with a history of hypertension presents to the clinic for a follow-up appointment. The patient has been taking a calcium channel blocker for blood pressure control and reports good adherence to the medication. During the visit, the patient mentions experiencing dizziness and occasional palpitations. The blood pressure reading is 140/88 mm Hg. What should the APRN consider as the next step in management?

Multiple Choice Answers: A) Increase the dosage of the calcium channel blocker. B) Switch to an ACE inhibitor. C) Prescribe a beta-blocker as an additional antihypertensive. D) Evaluate for possible arrhythmias. E) Recommend lifestyle modifications.

Correct Answer: D) Evaluate for possible arrhythmias.

Rationale: Dizziness and palpitations may be indicative of arrhythmias, which can be associated with certain antihypertensive medications like calcium channel blockers. It is essential to assess for arrhythmias before making further changes to the medication regimen and manage hypertension appropriately.

AANP Question 82: A 45-year-old patient with a history of hypertension presents to the primary care clinic for a routine follow-up. The patient has been taking a thiazide diuretic for blood pressure control and reports good adherence to the medication. The latest blood pressure reading is consistently elevated at 150/95 mm Hg. The patient has no other significant medical history or symptoms. What should the APRN do next?

Multiple Choice Answers: A) Increase the dosage of the thiazide diuretic. B) Prescribe an ACE inhibitor as an additional antihypertensive. C) Switch to an angiotensin receptor blocker (ARB). D) Recommend lifestyle modifications. E) Order a 24-hour ambulatory blood pressure monitoring (ABPM) test.

Correct Answer: E) Order a 24-hour ambulatory blood pressure monitoring (ABPM) test.

Rationale: In patients with persistent elevated blood pressure readings in the clinic, it is essential to rule out white coat hypertension and assess for blood pressure variability. Ordering a 24-hour ABPM test can provide a more accurate picture of the patient’s blood pressure patterns and guide treatment decisions.

AANP Question 83: A 55-year-old patient with newly diagnosed hypertension presents to the APRN for evaluation. The patient has no other significant medical history and is not currently taking any medications. The initial blood pressure reading is 160/100 mm Hg. What is the most appropriate initial intervention for this patient?

Multiple Choice Answers: A) Prescribe a thiazide diuretic as the first-line medication. B) Recommend lifestyle modifications and recheck blood pressure in 2 weeks. C) Start antihypertensive therapy with a calcium channel blocker. D) Order a renal artery ultrasound to assess for renovascular hypertension. E) Advise immediate hospitalization for blood pressure control.

Correct Answer: B) Recommend lifestyle modifications and recheck blood pressure in 2 weeks.

Rationale: Lifestyle modifications are often the initial intervention for newly diagnosed hypertension, especially when there are no significant comorbidities. Rechecking blood pressure after a period of lifestyle changes allows for assessment of their effectiveness before considering medication.

AANP Question 84: A 60-year-old patient with a history of hypertension presents to the clinic for a follow-up appointment. The patient has been taking an ACE inhibitor for blood pressure control and reports good adherence to the medication. During the visit, the patient mentions experiencing a persistent dry cough. The blood pressure reading is 140/88 mm Hg. What should the APRN consider as the next step in management?

Multiple Choice Answers: A) Increase the dosage of the ACE inhibitor. B) Switch to an angiotensin receptor blocker (ARB). C) Prescribe a calcium channel blocker as an additional antihypertensive. D) Refer the patient to a pulmonologist for further evaluation. E) Recommend lifestyle modifications.

Correct Answer: B) Switch to an angiotensin receptor blocker (ARB).

Rationale: A persistent dry cough is a known side effect of ACE inhibitors. In this scenario, switching the patient to an ARB is an appropriate course of action to manage hypertension while avoiding the side effect of cough associated with ACE inhibitors.

AANP Question 85: A 65-year-old patient with a history of hypertension presents to the primary care clinic for a follow-up visit. The patient has been taking an ACE inhibitor for blood pressure control and reports good adherence to the medication. However, the latest blood pressure reading is consistently elevated at 160/100 mm Hg. The patient has no other significant medical history or symptoms. What should the APRN do next?

Multiple Choice Answers: A) Increase the dosage of the ACE inhibitor. B) Switch to a thiazide diuretic as the first-line therapy. C) Add a calcium channel blocker to the current therapy. D) Recommend lifestyle modifications. E) Order a 24-hour ambulatory blood pressure monitoring (ABPM) test.

Correct Answer: E) Order a 24-hour ambulatory blood pressure monitoring (ABPM) test.

Rationale: When a patient’s blood pressure remains uncontrolled despite adherence to medication, it is important to assess for white coat hypertension or blood pressure variability. Ordering a 24-hour ABPM test can provide a more accurate evaluation of the patient’s blood pressure patterns and guide treatment decisions.

AANP Question 86: A 50-year-old patient with newly diagnosed hypertension presents to the APRN for evaluation. The patient has no other significant medical history and is not currently taking any medications. The initial blood pressure reading is 160/100 mm Hg. What is the most appropriate initial intervention for this patient?

Multiple Choice Answers: A) Prescribe a thiazide diuretic as the first-line medication. B) Recommend lifestyle modifications and recheck blood pressure in 2 weeks. C) Start antihypertensive therapy with an ACE inhibitor. D) Order a renal artery ultrasound to assess for renovascular hypertension. E) Advise immediate hospitalization for blood pressure control.

Correct Answer: B) Recommend lifestyle modifications and recheck blood pressure in 2 weeks.

Rationale: Lifestyle modifications are often the initial intervention for newly diagnosed hypertension, especially when there are no significant comorbidities. Rechecking blood pressure after implementing lifestyle changes allows for assessment of their effectiveness before considering medication.

AANP Question 87: A 55-year-old patient with a history of hypertension presents for a routine check-up. The patient is currently taking an ACE inhibitor for blood pressure control and reports good adherence to the medication. The latest blood pressure reading is 142/90 mm Hg. During the visit, the patient mentions experiencing persistent dry cough. What should the APRN consider as the next step in management?

Multiple Choice Answers: A) Increase the dosage of the ACE inhibitor. B) Switch to an angiotensin receptor blocker (ARB). C) Prescribe a calcium channel blocker as an additional antihypertensive. D) Refer the patient to a pulmonologist for further evaluation. E) Recommend lifestyle modifications.

Correct Answer: B) Switch to an angiotensin receptor blocker (ARB).

Rationale: A persistent dry cough is a common side effect associated with ACE inhibitors. In this scenario, switching the patient to an ARB is a reasonable approach to manage hypertension while avoiding the side effect of cough associated with ACE inhibitors

AANP Question 88: A 65-year-old patient with a history of hypertension, diabetes, and hyperlipidemia presents to the primary care clinic for a routine check-up. The patient reports taking prescribed medications regularly. During the assessment, the patient mentions experiencing sudden weakness on one side of the face and difficulty speaking. What is the APRN’s most appropriate action?

Multiple Choice Answers: A) Perform a comprehensive neurological examination. B) Order an immediate brain MRI to assess for stroke. C) Recommend lifestyle modifications to manage risk factors. D) Refer the patient to a physical therapist for rehabilitation. E) Prescribe a blood pressure-lowering medication.

Correct Answer: A) Perform a comprehensive neurological examination.

Rationale: When a patient presents with sudden neurological symptoms suggestive of a stroke, the initial action should involve a comprehensive neurological examination to assess the extent and severity of the symptoms. This examination helps determine the urgency of further diagnostic tests or interventions.

AANP Question 89: A 55-year-old patient with a history of hypertension and smoking presents to the emergency department with sudden-onset severe headache, vomiting, and confusion. On examination, the patient has a decreased level of consciousness. What diagnostic test is most appropriate for confirming a suspected hemorrhagic stroke?

Multiple Choice Answers: A) Non-contrast head CT scan. B) Brain MRI with contrast. C) Carotid artery ultrasound. D) Electroencephalogram (EEG). E) Lumbar puncture (LP).

Correct Answer: A) Non-contrast head CT scan.

Rationale: A non-contrast head CT scan is the preferred initial diagnostic test for confirming a suspected hemorrhagic stroke. It helps identify the presence and location of bleeding within the brain, guiding further treatment decisions.

AANP Question 90: A 70-year-old patient with a history of atrial fibrillation (AF) and anticoagulation therapy presents to the clinic for a follow-up appointment. The patient reports compliance with prescribed anticoagulant medication. However, during the assessment, the patient mentions experiencing sudden weakness and numbness on one side of the body. What is the APRN’s most appropriate initial action?

Multiple Choice Answers: A) Administer a thrombolytic agent to dissolve a possible clot. B) Order a brain MRI with contrast to assess for ischemic stroke. C) Adjust the dosage of the anticoagulant medication. D) Refer the patient to a neurologist for further evaluation. E) Assess for signs of gastrointestinal bleeding.

Correct Answer: B) Order a brain MRI with contrast to assess for ischemic stroke.

Rationale: In a patient with atrial fibrillation on anticoagulation therapy, the initial action when experiencing sudden neurological symptoms should involve assessing for the presence of an ischemic stroke. A brain MRI with contrast is a valuable diagnostic tool for this purpose. Administering thrombolytic agents would not be appropriate without confirmation of an ischemic stroke.

AANP Question 91: A 68-year-old patient with a history of hypertension, atrial fibrillation, and diabetes presents to the emergency department with sudden-onset weakness on one side of the body, slurred speech, and difficulty raising one arm. The symptoms began approximately 45 minutes ago. What is the APRN’s most appropriate action in this acute scenario?

Multiple Choice Answers: A) Administer tissue plasminogen activator (tPA) for thrombolysis. B) Order a brain CT scan to assess for hemorrhagic stroke. C) Start the patient on anticoagulant therapy immediately. D) Consult a neurologist for further evaluation. E) Perform a carotid ultrasound to assess for stenosis.

Correct Answer: A) Administer tissue plasminogen activator (tPA) for thrombolysis.

Rationale: In a patient presenting with acute stroke symptoms, the administration of tPA within the first few hours after symptom onset can significantly improve outcomes in ischemic stroke. Time is critical in such cases, and rapid action is needed to assess eligibility and administer tPA when appropriate.

AANP Question 92: A 60-year-old patient with a history of smoking and hypertension visits the primary care clinic for a routine check-up. The patient has been taking antihypertensive medication as prescribed. During the examination, the APRN detects a carotid bruit on auscultation. What is the most appropriate next step?

Multiple Choice Answers: A) Immediately refer the patient to a vascular surgeon. B) Order a carotid ultrasound to assess for stenosis. C) Increase the dosage of the antihypertensive medication. D) Start the patient on an antiplatelet medication. E) Recommend dietary changes to reduce cholesterol intake.

Correct Answer: B) Order a carotid ultrasound to assess for stenosis.

Rationale: The presence of a carotid bruit may suggest carotid artery stenosis, which is a risk factor for ischemic stroke. Ordering a carotid ultrasound is an appropriate next step to assess the degree of stenosis and guide further management decisions.

AANP Question 93: A 75-year-old patient with a history of atrial fibrillation (AF) presents to the clinic for a follow-up appointment. The patient has been prescribed anticoagulation therapy with warfarin to reduce the risk of stroke. However, the patient mentions difficulty maintaining a consistent INR (international normalized ratio) within the therapeutic range. What is the APRN’s most appropriate action?

Multiple Choice Answers: A) Switch the patient to a direct oral anticoagulant (DOAC). B) Increase the warfarin dosage to achieve a stable INR. C) Recommend dietary changes to increase vitamin K intake. D) Refer the patient to a hematologist for further evaluation. E) Discontinue anticoagulation therapy due to the INR fluctuations.

Correct Answer: A) Switch the patient to a direct oral anticoagulant (DOAC).

Rationale: In patients with difficulty maintaining a stable INR on warfarin, switching to a DOAC may be a more suitable option for anticoagulation therapy. DOACs have a more predictable anticoagulant effect and do not require frequent INR monitoring, making them a convenient choice for many patients with AF.

AANP Question 94: A 62-year-old patient with a history of hypertension and smoking presents to the primary care clinic with a complaint of sudden-onset temporary vision loss in one eye, which lasted for about 10 minutes. The patient reports no other symptoms and feels completely normal now. What is the most appropriate initial assessment for the APRN?

Multiple Choice Answers: A) Order a complete blood count (CBC) to assess for anemia. B) Perform a carotid artery ultrasound to evaluate for stenosis. C) Administer aspirin and refer the patient for neuroimaging. D) Schedule an urgent referral to an ophthalmologist. E) Recommend lifestyle modifications to manage hypertension.

Correct Answer: C) Administer aspirin and refer the patient for neuroimaging.

Rationale: The patient’s complaint of sudden-onset temporary vision loss is suggestive of amaurosis fugax, which can be a symptom of a transient ischemic attack (TIA). Administering aspirin and referring the patient for neuroimaging (e.g., MRI or CT) is essential to assess for the presence of vascular issues in the brain and initiate appropriate management.

AANP Question 95: A 55-year-old patient with a history of diabetes and hyperlipidemia presents to the emergency department with sudden-onset weakness on one side of the body and slurred speech, which lasted for approximately 30 minutes and has now resolved. The patient denies any recent head injury. What is the APRN’s most appropriate action?

Multiple Choice Answers: A) Admit the patient for observation and initiate anticoagulation. B) Perform a carotid artery ultrasound to assess for stenosis. C) Administer tissue plasminogen activator (tPA) for thrombolysis. D) Order a head CT scan to evaluate for bleeding or ischemia. E) Recommend outpatient follow-up with a neurologist.

Correct Answer: D) Order a head CT scan to evaluate for bleeding or ischemia.

Rationale: The patient’s presentation is suggestive of a possible transient ischemic attack (TIA) or a minor stroke. The initial diagnostic step should be a head CT scan to assess for bleeding or ischemia, which will guide further management decisions.

AANP Question 96: A 70-year-old patient with a history of atrial fibrillation (AF) presents to the clinic for a follow-up appointment. The patient is taking anticoagulation therapy with warfarin to reduce the risk of stroke. During the visit, the patient mentions experiencing sudden-onset weakness on one side of the body and slurred speech that lasted for approximately 10 minutes but has now resolved. What is the APRN’s most appropriate action?

Multiple Choice Answers: A) Administer tissue plasminogen activator (tPA) for thrombolysis. B) Increase the warfarin dosage to achieve a higher INR. C) Order a carotid artery ultrasound to assess for stenosis. D) Refer the patient for cardiac ablation to manage AF. E) Initiate dual antiplatelet therapy with aspirin and clopidogrel.

Correct Answer: C) Order a carotid artery ultrasound to assess for stenosis.

Rationale: In a patient with a history of AF who presents with symptoms suggestive of a possible TIA, it is important to assess for other potential causes, including carotid artery stenosis. Ordering a carotid artery ultrasound is an appropriate step to evaluate the vascular status and guide further management.

AANP Question 97: A 60-year-old patient with a history of hypertension and diabetes presents to the primary care clinic complaining of sudden-onset weakness on one side of the body and difficulty speaking, which lasted for about 20 minutes but has now resolved. The patient denies any recent head injury. What is the most appropriate initial action for the APRN?

Multiple Choice Answers: A) Administer aspirin and admit the patient for observation. B) Perform a carotid artery ultrasound to assess for stenosis. C) Order a head CT scan to evaluate for bleeding or ischemia. D) Schedule an urgent referral to a neurologist. E) Recommend lifestyle modifications to manage diabetes.

Correct Answer: C) Order a head CT scan to evaluate for bleeding or ischemia.

Rationale: The patient’s symptoms are suggestive of a possible transient ischemic attack (TIA) or a minor stroke. The initial diagnostic step should be a head CT scan to assess for bleeding or ischemia, which will guide further management decisions.

AANP Question 98: A 55-year-old patient with a history of hyperlipidemia and smoking presents to the emergency department with sudden-onset temporary vision loss in one eye, which lasted for about 15 minutes. The patient reports no other symptoms and feels completely normal now. What is the APRN’s most appropriate initial assessment?

Multiple Choice Answers: A) Order a complete blood count (CBC) to assess for anemia. B) Perform a carotid artery ultrasound to evaluate for stenosis. C) Administer aspirin and refer the patient for neuroimaging. D) Schedule an urgent referral to an ophthalmologist. E) Recommend lifestyle modifications to manage hyperlipidemia.

Correct Answer: B) Perform a carotid artery ultrasound to evaluate for stenosis.

Rationale: Sudden-onset temporary vision loss in one eye, known as amaurosis fugax, can be associated with carotid artery stenosis, a risk factor for transient ischemic attacks (TIAs). Performing a carotid artery ultrasound is an appropriate initial assessment to evaluate for vascular issues.

AANP Question 99: A 70-year-old patient with a history of atrial fibrillation (AF) presents to the clinic for a follow-up appointment. The patient is taking anticoagulation therapy with a direct oral anticoagulant (DOAC) to reduce the risk of stroke. During the visit, the patient mentions experiencing sudden-onset weakness on one side of the body and slurred speech that lasted for approximately 10 minutes but has now resolved. What is the APRN’s most appropriate action?

Multiple Choice Answers: A) Administer tissue plasminogen activator (tPA) for thrombolysis. B) Increase the dosage of the DOAC to achieve a higher anticoagulant effect. C) Order a carotid artery ultrasound to assess for stenosis. D) Refer the patient for cardiac ablation to manage AF. E) Initiate dual antiplatelet therapy with aspirin and clopidogrel.

Correct Answer: C) Order a carotid artery ultrasound to assess for stenosis.

Rationale: In a patient with a history of AF who presents with symptoms suggestive of a possible TIA, it is important to assess for other potential causes, including carotid artery stenosis. Ordering a carotid artery ultrasound is an appropriate step to evaluate the vascular status and guide further management.

AANP Question 100: A 45-year-old patient with a history of hypertension, obesity, and a sedentary lifestyle presents to the primary care clinic for a routine check-up. The patient’s blood pressure is consistently above 140/90 mm Hg, and the body mass index (BMI) is 32. What is the most significant modifiable risk factor for stroke in this patient?

Multiple Choice Answers: A) Age B) Hypertension C) Obesity D) Sedentary lifestyle E) Family history

Correct Answer: B) Hypertension

Rationale: While age and family history are non-modifiable risk factors for stroke, hypertension is one of the most significant modifiable risk factors. Controlling blood pressure through lifestyle modifications and medication can substantially reduce the risk of stroke.

AANP Question 101: A 60-year-old patient with a history of atrial fibrillation (AF) presents to the clinic for a follow-up appointment. The patient is taking an anticoagulant medication to reduce the risk of stroke. Which of the following factors increases the risk of stroke in patients with AF?

Multiple Choice Answers: A) Regular exercise B) Well-controlled blood pressure C) Low cholesterol levels D) Age over 75 E) Adequate hydration

Correct Answer: D) Age over 75

Rationale: Age over 75 is a significant risk factor for stroke in patients with atrial fibrillation (AF). Other factors such as hypertension, prior stroke or TIA, and heart failure also contribute to increased stroke risk in AF patients.

AANP Question 102: A 55-year-old patient with a history of smoking and a family history of stroke presents for a routine check-up. The patient quit smoking three years ago but has a 20-pack-year smoking history. Which of the following factors contributes most to the patient’s ongoing risk of stroke?

Multiple Choice Answers: A) Family history of stroke B) Smoking history C) Age D) Diet high in saturated fats E) Regular exercise routine

Correct Answer: B) Smoking history

Rationale: While family history, age, and diet play roles in stroke risk, smoking is a major modifiable risk factor that continues to contribute significantly to stroke risk even after quitting. Smokers have an increased risk of stroke, and the risk decreases gradually after cessation but may remain elevated for some time.

AANP Question 103: A 68-year-old patient with chronic obstructive pulmonary disease (COPD) presents to the emergency department with increased shortness of breath and altered mental status. On physical examination, the patient is drowsy and has shallow breathing. Which arterial blood gas (ABG) parameter is most likely to be abnormal in this patient?

Multiple Choice Answers: A) pH B) PaO2 (partial pressure of oxygen) C) HCO3- (bicarbonate) D) PaCO2 (partial pressure of carbon dioxide) E) SaO2 (arterial oxygen saturation)

Correct Answer: D) PaCO2 (partial pressure of carbon dioxide)

Rationale: In a patient with COPD experiencing increased shortness of breath and altered mental status, the primary ABG abnormality is typically an elevated PaCO2 due to impaired ventilation and the inability to eliminate carbon dioxide effectively.

AANP Question 104: A 45-year-old patient with uncontrolled diabetes presents to the clinic with complaints of deep, rapid breathing and fruity breath odor. The patient appears confused and is breathing at a rate of 30 breaths per minute. Which ABG parameter is most likely to be abnormal in this patient?

Multiple Choice Answers: A) pH B) PaO2 (partial pressure of oxygen) C) HCO3- (bicarbonate) D) PaCO2 (partial pressure of carbon dioxide) E) SaO2 (arterial oxygen saturation)

Correct Answer: A) pH

Rationale: The patient’s deep, rapid breathing, fruity breath odor, and confusion are indicative of diabetic ketoacidosis (DKA), which is associated with metabolic acidosis. In DKA, the primary ABG abnormality is a decreased pH.

AANP Question 105: A 60-year-old patient with a history of heart failure presents to the cardiology clinic for a follow-up appointment. The patient is currently taking diuretics, ACE inhibitors, and beta-blockers. Which ABG parameter is most likely to be affected by this medication regimen?

Multiple Choice Answers: A) pH B) PaO2 (partial pressure of oxygen) C) HCO3- (bicarbonate) D) PaCO2 (partial pressure of carbon dioxide) E) SaO2 (arterial oxygen saturation)

Correct Answer: D) PaCO2 (partial pressure of carbon dioxide)

Rationale: Diuretics can lead to fluid loss and potentially affect electrolyte balance, which may influence the patient’s acid-base status. In the context of heart failure treatment, diuretics can cause a decrease in plasma volume, leading to an increase in PaCO2 due to compensatory hypoventilation. This can result in respiratory acidosis.

AANP Question 106: A 65-year-old patient presents to the primary care clinic with a complaint of leg pain that occurs during walking and is relieved with rest. The pain is typically described as cramping in the calves and occurs after walking short distances. What term best describes this symptom?

Multiple Choice Answers: A) Resting pain B) Intermittent claudication C) Neuropathic pain D) Ischemic pain E) Chronic fatigue

Correct Answer: B) Intermittent claudication

Rationale: Intermittent claudication is characterized by leg pain that occurs during physical activity, such as walking, and is relieved with rest. This is a classic symptom of peripheral artery disease (PAD), often due to reduced blood flow to the leg muscles during exercise.

AANP Question 107: A 55-year-old patient with a history of smoking and hypertension presents with complaints of leg pain and cramping while walking. The pain is consistently present, even at rest, and is not relieved by cessation of physical activity. What term best describes this symptom?

Multiple Choice Answers: A) Resting pain B) Intermittent claudication C) Neuropathic pain D) Ischemic pain E) Chronic fatigue

Correct Answer: A) Resting pain

Rationale: Resting pain in the legs is a concerning symptom that may indicate severe ischemia and inadequate blood flow to the limbs, typically seen in advanced peripheral artery disease (PAD). Unlike intermittent claudication, resting pain is not relieved by rest.

AANP Question 108: A 70-year-old patient with a history of diabetes and obesity presents to the clinic with complaints of pain and cramping in the legs while walking. The pain is often described as burning or shooting and is accompanied by numbness and tingling. What term best describes this symptom?

Multiple Choice Answers: A) Resting pain B) Intermittent claudication C) Neuropathic pain D) Ischemic pain E) Chronic fatigue

Correct Answer: C) Neuropathic pain

Rationale: Neuropathic pain in the legs, often accompanied by symptoms like numbness and tingling, is suggestive of peripheral neuropathy, which can occur in patients with diabetes. It is different from intermittent claudication, which is primarily due to reduced blood flow and manifests as cramping during walking.

AANP Question 109: A 60-year-old patient with a history of smoking and diabetes mellitus presents to the vascular clinic with a complaint of constant, severe pain in the toes and forefoot that worsens at night and is not relieved with rest. The patient also notes changes in skin color and temperature, including cool and pale skin. What term best describes this symptom?

Multiple Choice Answers: A) Intermittent claudication B) Neuropathic pain C) Rest pain D) Ischemic pain E) Musculoskeletal pain

Correct Answer: C) Rest pain

Rationale: Rest pain is characterized by continuous pain in the extremities, typically the lower legs or feet, that occurs at rest and is not relieved by rest. It is a hallmark symptom of advanced peripheral artery disease (PAD) and is often associated with severe ischemia.

AANP Question 110: A 65-year-old patient with a history of hypertension and hyperlipidemia presents with complaints of leg discomfort and cramping during walking, which is relieved by stopping and resting. The patient denies experiencing pain at night or while at rest. What term best describes this symptom?

Multiple Choice Answers: A) Intermittent claudication B) Neuropathic pain C) Rest pain D) Ischemic pain E) Musculoskeletal pain

Correct Answer: A) Intermittent claudication

Rationale: Intermittent claudication is characterized by leg pain or discomfort during physical activity, such as walking, that is relieved by stopping and resting. This symptom is often associated with peripheral artery disease (PAD) and reduced blood flow during exercise.

AANP Question 111: A 70-year-old patient with a history of heart disease presents with complaints of persistent leg pain, particularly in the feet, which occurs when sitting or lying down and is relieved by dangling the legs over the bedside. The patient reports having to hang the legs over the edge of the bed for relief. What term best describes this symptom?

Multiple Choice Answers: A) Intermittent claudication B) Neuropathic pain C) Rest pain D) Ischemic pain E) Musculoskeletal pain

Correct Answer: C) Rest pain

Rationale: Rest pain is characterized by continuous pain that occurs at rest, often when the legs are elevated, and is relieved by hanging the legs down. This symptom is indicative of severe peripheral artery disease (PAD) and inadequate blood flow to the lower extremities.

Question 112: A 58-year-old patient with a history of coronary artery disease (CAD) presents to the cardiology clinic with recurrent angina (chest pain) despite optimal medical therapy. The patient’s coronary angiogram reveals a significant blockage in a major coronary artery. What is the most appropriate next step in the management of this patient?

Multiple Choice Answers: A) Continue medical therapy and monitor the patient’s symptoms. B) Perform a stress test to assess the extent of myocardial ischemia. C) Refer the patient for coronary artery bypass grafting (CABG) surgery. D) Recommend percutaneous coronary intervention (PCI) with angioplasty and stent placement. E) Increase the dosage of antianginal medications.

Correct Answer: D) Recommend percutaneous coronary intervention (PCI) with angioplasty and stent placement.

Rationale: In a patient with recurrent angina despite optimal medical therapy and significant coronary artery blockage, the most appropriate next step is often PCI with angioplasty and stent placement to improve coronary blood flow and relieve symptoms.

AANP Question 113: A 62-year-old patient with a history of hypertension and stable angina has undergone successful PCI with stent placement in a coronary artery. The patient is currently on dual antiplatelet therapy (aspirin and clopidogrel). What is the recommended duration of dual antiplatelet therapy following PCI with stent placement?

Multiple Choice Answers: A) 1 week B) 1 month C) 3 months D) 6 months E) 12 months

Correct Answer: E) 12 months

Rationale: Following PCI with stent placement, guidelines typically recommend dual antiplatelet therapy (aspirin and a P2Y12 inhibitor like clopidogrel) for a duration of 12 months to reduce the risk of stent thrombosis and cardiovascular events.

AANP Question 114: A 50-year-old patient presents to the emergency department with acute ST-segment elevation myocardial infarction (STEMI). After prompt diagnosis, the patient is taken to the catheterization lab for primary PCI. During the procedure, the interventional cardiologist successfully opens the occluded coronary artery with angioplasty and places a drug-eluting stent. What is a potential long-term complication associated with drug-eluting stents?

Multiple Choice Answers: A) Hypertension B) Stent thrombosis C) Bradycardia D) Peripheral artery disease E) Gastroesophageal reflux disease (GERD)

Correct Answer: B) Stent thrombosis

Rationale: Drug-eluting stents, while effective in reducing restenosis rates, can pose a risk of late stent thrombosis, which is a potential long-term complication. Patients receiving drug-eluting stents require prolonged dual antiplatelet therapy to mitigate this risk.

AANP Question 115: A 70-year-old patient with a history of peripheral artery disease (PAD) presents to the vascular clinic with severe claudication and pain at rest in the lower extremities. Non-invasive vascular studies reveal a long-segment occlusion in the femoral-popliteal artery. What is the most appropriate treatment option for this patient?

Multiple Choice Answers: A) Continue with conservative management and lifestyle modifications. B) Prescribe cilostazol (Pletal) as a first-line therapy. C) Refer the patient for a femoral-popliteal bypass surgery. D) Perform percutaneous transluminal angioplasty (PTA) with stent placement. E) Administer anticoagulant therapy to prevent further occlusion.

Correct Answer: C) Refer the patient for a femoral-popliteal bypass surgery.

Rationale: In cases of severe femoral-popliteal artery occlusion with claudication and pain at rest, surgical intervention such as femoral-popliteal bypass surgery is often the preferred treatment option to improve blood flow to the lower extremities.

AANP Question 116: A 55-year-old patient with a history of diabetes and hypertension presents with recurrent lower extremity ulcers that have not healed despite wound care and medical management. Non-invasive vascular studies reveal significant femoral-popliteal artery disease with impaired blood flow. What is the most appropriate intervention for this patient?

Multiple Choice Answers: A) Continue wound care and adjust diabetes management. B) Prescribe an oral vasodilator medication. C) Refer the patient for femoral-popliteal bypass surgery. D) Attempt endovascular therapy with angioplasty and stent placement. E) Administer topical antibiotics for ulcer treatment.

Correct Answer: C) Refer the patient for femoral-popliteal bypass surgery.

Rationale: In cases of chronic lower extremity ulcers with significant femoral-popliteal artery disease, femoral-popliteal bypass surgery is often necessary to restore adequate blood flow and promote wound healing.

AANP Question 117: A 60-year-old patient with a history of smoking presents to the vascular clinic with claudication and pain in the right leg during walking. Non-invasive vascular studies confirm a focal stenosis in the femoral-popliteal artery. What is the initial treatment of choice for this patient?

Multiple Choice Answers: A) Immediate referral for femoral-popliteal bypass surgery. B) Initiation of dual antiplatelet therapy (aspirin and clopidogrel). C) Attempt percutaneous transluminal angioplasty (PTA) with or without stent placement. D) Prescribe a statin medication for cholesterol management. E) Recommendation of lifestyle modifications and supervised exercise.

Correct Answer: C) Attempt percutaneous transluminal angioplasty (PTA) with or without stent placement.

Rationale: In cases of focal stenosis in the femoral-popliteal artery causing claudication, the initial approach often involves attempting endovascular therapy such as PTA with or without stent placement to improve blood flow and relieve symptoms. Surgery is usually considered if endovascular intervention is not successful.

Rationale: This question evaluates the nurse practitioner’s knowledge of the initial screening test for assessing adrenal function. Measurement of serum cortisol levels is the primary diagnostic step in evaluating adrenal disorders, including conditions like Cushing’s syndrome or adrenal insufficiency.